- Fri Aug 02, 2013 11:00 pm
#40627
Complete Question Explanation
(The complete setup for this game can be found here: lsat/viewtopic.php?t=14136)
The correct answer choice is (D)
In Global, Could Be True questions you must typically rely on your main diagram, and this question is no different. However, instead of eliminating answer choices using Not Laws, this question often requires you to rely on the fourth rule to eliminate answer choices. With that in mind, let’s examine each answer choice individually:
Answer choice (A): This answer cannot occur because G is the third school to which snacks are delivered, and F must be the first or second school to which snacks are delivered.
Answer choice (B): This answer choice cannot occur because if snacks are delivered to G before they are delivered to I, then because snacks are delivered to G third, the snacks delivery to I would occur fourth. But, from the fourth rule, then the juices would be delivered first to I, meaning that I’s juices delivery would be earlier than G’s juices delivery.
Answer choice (C): This answer choice cannot occur because either H or I must be the first school to which juices are delivered and the fourth school to which snacks are delivered, meaning that one of H and I is always delivered before the other in the juices, and then delivered after the other in the snacks.
Answer choice (D): This is the correct answer choice. Here’s one hypothetical that shows the scenario in this answer choice could occur:
Answer choice (E): This answer choice is functionally the same as answer choice (C). Either H or I must be the first school to which juices are delivered and the fourth school to which snacks are delivered, meaning that one of H and I is always delivered before the other in the juices, and then delivered after the other in the snacks.
(The complete setup for this game can be found here: lsat/viewtopic.php?t=14136)
The correct answer choice is (D)
In Global, Could Be True questions you must typically rely on your main diagram, and this question is no different. However, instead of eliminating answer choices using Not Laws, this question often requires you to rely on the fourth rule to eliminate answer choices. With that in mind, let’s examine each answer choice individually:
Answer choice (A): This answer cannot occur because G is the third school to which snacks are delivered, and F must be the first or second school to which snacks are delivered.
Answer choice (B): This answer choice cannot occur because if snacks are delivered to G before they are delivered to I, then because snacks are delivered to G third, the snacks delivery to I would occur fourth. But, from the fourth rule, then the juices would be delivered first to I, meaning that I’s juices delivery would be earlier than G’s juices delivery.
Answer choice (C): This answer choice cannot occur because either H or I must be the first school to which juices are delivered and the fourth school to which snacks are delivered, meaning that one of H and I is always delivered before the other in the juices, and then delivered after the other in the snacks.
Answer choice (D): This is the correct answer choice. Here’s one hypothetical that shows the scenario in this answer choice could occur:
Answer choice (E): This answer choice is functionally the same as answer choice (C). Either H or I must be the first school to which juices are delivered and the fourth school to which snacks are delivered, meaning that one of H and I is always delivered before the other in the juices, and then delivered after the other in the snacks.
You do not have the required permissions to view the files attached to this post.